Sie sind auf Seite 1von 18

SECTION SEVEN

Neurology
C H A P T E R 91 

Stroke
Todd J. Crocco | William J. Meurer

PRINCIPLES bifurcation. A marked reduction in flow results when the stenosis


occludes more than 90% of the blood vessel diameter. With
Background further ulceration and thrombosis, platelets adhere to the region.
A clot then either embolizes or occludes the artery.
Stroke is the fifth leading cause of death in the United States and Lacunae, or small-vessel strokes, involve small terminal sec-
a leading cause of long-term disability.1 It affects about 795,000 tions of the vasculature and more commonly occur in patients
people per year. On average, someone has a stroke every 40 with diabetes and hypertension. A history of hypertension is
seconds, and someone dies of a stroke every 4 minutes.2 Stroke present in 80% to 90% of patients who experience lacunar strokes.
patients have an in-hospital mortality rate of 5% to 10% for isch- The subcortical areas of the cerebrum and brainstem often are
emic stroke and 40% to 60% for intracerebral hemorrhage (ICH).3 involved. The infarcts range in size from a few millimeters to 2 cm
Only 10% of stroke survivors will recover completely, making and are seen most commonly in the basal ganglia, thalamus, pons,
stroke a leading cause of adult disability. and internal capsule. They may be caused by small emboli or by
Stroke can be defined as any vascular injury that reduces cere- a process termed lipohyalinosis, which occurs in patients with
bral blood flow (CBF) to a specific region of the brain, retina, or hypertensive cerebral vasculopathy.
spinal cord, causing neurologic impairment. The onset of symp- One-fourth of all ischemic strokes are cardioembolic in nature.
toms may be sudden or stuttering, often with transient or perma- Embolization of a mural thrombus in patients with atrial fibrilla-
nent loss of neurologic function. Approximately 87% of all strokes tion is the most common mechanism, and patients with atrial
are ischemic in origin, caused by the occlusion of a cerebral vessel. fibrillation have an approximate fivefold increased risk for devel-
Approximately 13% are hemorrhagic strokes caused by the opment of a stroke. Noncardiac sources of emboli may include
rupture of a blood vessel into the parenchyma of the brain (ICH) diseased portions of extracranial arteries, resulting in an artery-
or into the subarachnoid space (subarachnoid hemorrhage to-artery embolus. One common example is amaurosis fugax, in
[SAH]). Only ischemic stroke involving the brain and ICH are which emboli from a proximal carotid artery plaque embolizes to
discussed in this chapter. SAH is discussed in Chapter 93. the ophthalmic artery, causing transient monocular blindness.
Prior to the reperfusion era, treatment for stroke was not Although stroke risk increases with age, approximately 3% to
focused on reversal of damage and consisted of stabilization, 4% of all strokes occur in patients 15 to 45 years old, and there
observation, and rehabilitation. Current acute interventional have been trends observed showing the average age of first stroke
treatment regimens are designed to reverse or minimize brain is becoming younger.5 Although atherosclerosis is the most
damage.4 Strategies include blood pressure (BP) management, common cause in elders, causative disorders and conditions in
anticoagulation, thrombolytic therapy, catheter-based interven- younger patients often are uncommon and may be reversible.
tions, and surgery. Pregnancy, the use of oral contraceptives, antiphospholipid
antibodies (such as, lupus anticoagulant and anticardiolipin anti-
Epidemiology bodies), protein S and C deficiencies, sickle cell anemia, and poly-
cythemia all predispose patients to sludging or thrombosis,
Ischemic Stroke thereby increasing the risk of stroke. Fibromuscular dysplasia of
the cerebral vasculature also may lead to stroke, and in rare
An estimated 610,000 “first-ever” ischemic strokes occur each year instances prolonged vasoconstriction from a migraine syndrome
in the United States. These may result from either in situ throm- causes stroke. Recreational drugs such as cocaine, phenylpropa-
bosis or embolic obstruction from a more proximal source, nolamine, and amphetamines are potent vasoconstrictors that
usually the heart. In more than one-third of these first-ever have been associated with both ischemic and hemorrhagic stroke.
strokes, no clear cause is identified. Strokes of all subtypes are Infectious processes, particularly varicella and recently fungal
more common in African American and Hispanics versus non- meningitis, can induce vasculopathies that lead to stroke as well
Hispanic whites. or can induce longer-term inflammatory processes that ultimately
Approximately one-third of all ischemic strokes are throm- cause a clinical stroke.
botic in nature. These can be caused by either large- or small- Carotid and vertebral dissections often are associated with
vessel occlusions. Common areas for large-vessel occlusions are trauma but may follow mild events such as sneezing. Dissections
cerebral vessel branch points, especially in the distribution of are the leading determined cause of stroke in the young and are
the internal carotid artery. Thrombosis usually results from clot slightly less common than idiopathic strokes. Carotid and verte-
formation in the area of an ulcerated atherosclerotic plaque bral dissections also are seen more frequently in people with
that forms in the area of turbulent blood flow, such as a vessel underlying pathology of the vessel wall, such as in fibromuscular
1241
Downloaded for Theo Salean (theosalean@gmail.com) at Sriwijaya University from ClinicalKey.com by Elsevier on January 17, 2020.
For personal use only. No other uses without permission. Copyright ©2020. Elsevier Inc. All rights reserved.
1242 PART III  Medicine and Surgery  |  SECTION Seven  Neurology

dysplasia and connective tissue disorders. Alteration in the vessel BOX 91.1 
intima can lead to vessel stenosis, occlusion, or embolism. The
patient may report a minor preceding event, such as spinal manip- Most Common Sites for Hypertensive
ulation, yoga, working overhead, coughing, or vomiting. Present-
ing manifestations may include headache, facial pain, visual Intracranial Hemorrhage
changes, cranial nerve (CN) palsies, pain over the affected vessel,
Horner’s syndrome, amaurosis fugax, SAH, or an ischemic stroke. AFFECTED AREA (FREQUENCY)
The headache frequently is unilateral and may occur days before Putamen (44%)
onset of the other neurologic symptoms. Dissections are typically Thalamus (13%)
diagnosed by noninvasive modalities, such as ultrasonography, Cerebellum (9%)
magnetic resonance angiography (MRA), and computed tomog- Pons (9%)
Other cortical areas (25%)
raphy angiography (CTA). Medical therapy options include early
anticoagulation if SAH/intracranial dissection is not suspected.
COMMON CLINICAL PRESENTATION
The existing data comparing antiplatelet treatment to anticoagu- Contralateral motor/sensory loss
lation is generally limited and antiplatelet treatment is generally Limb pain, speech difficulty
simpler and safer.6 The use of tissue plasminogen activator (tPA) Uncoordinated movements of trunk and limbs
is considered as a safe and effective in extracranial carotid or Numbness, weakness, ataxia, dizziness
vertebral dissection patients as in any other eligible patient.7 Numbness, weakness, language disturbances

Transient Ischemic Attack


A transient ischemic attack (TIA) was historically defined as a
neurologic deficit with complete resolution within 24 hours;
however, a portion of TIA cases have evidence of permanent brain are immediate alterations in CBF and extensive changes in cellular
ischemia on neuroimaging. Therefore, the American Heart Asso- homeostasis. The normal CBF is approximately 40 to 60 mL/100 g
ciation (AHA) has adopted a tissue-based definition: A transient of brain per minute. When CBF drops below 15 to 18 mL/100 g
episode of neurologic dysfunction caused by focal brain, spinal of brain per minute, several physiologic changes occur. The brain
cord, or retinal ischemia, without acute infarction. loses electrical activity, becoming electrically “silent,” although
About 240,000 TIAs per year occur in the United States, with neuronal membrane integrity and function remain intact. Clini-
an incidence rate of 8 per 1000 person years. TIAs constitute an cally, the areas of the brain maintaining electrical silence manifest
important warning sign for the future development of cerebral a neurologic deficit, even though the brain cells are viable. When
infarction. Approximately 10% of the patients who experience a CBF is below 10 mL/100 g of brain per minute, membrane failure
TIA will experience a stroke within 3 months of the sentinel event, occurs, with a subsequent increase in the extracellular potassium
and one-half of these occur within the first 2 days. and intracellular calcium and eventual cell death.
The ischemic penumbra is the area of the brain surrounding
Hemorrhagic Stroke the primary injury, which is preserved by a tenuous supply of
blood from collateral vessels. This border zone of neuronal tissue
Spontaneous ICH causes 10% to 15% of all acute strokes, affecting is the area of greatest interest to investigators for possible salvage
approximately 65,000 patients per year. It carries a 30-day mortal- in both ischemic and hemorrhagic stroke. In ischemic stroke, the
ity rate of up to 50% with one-half of patients dying in the first duration of occlusion plays a critical role in neuronal survival.
2 days. Among survivors, only one in five are living independently In ICH, acute vessel rupture is most often caused by underly-
at 6 months. ing small vessel disease and causes injury by several mechanisms.
The two major underlying causes of ICH are hypertensive First, there is mass effect from the hematoma itself, followed
vasculopathy (caused by long-standing hypertension) and cere- by activation of the coagulation cascade, release of inflamma-
bral amyloid angiopathy (usually found in elders, which is the tory cytokines, and blood-brain barrier (BBB) disruption. This
result of amyloid deposition in cerebral vessel walls). Hypertensive leads to perihematomal edema formation and secondary brain
hemorrhage results from degenerative changes in the small pen- injury. Finally, continued bleeding, or hematoma expansion,
etrating arteries and arterioles, leading to lipohyalinosis of small, occurs in many patients—either continued bleeding from the
deep penetrating arteries. Such hemorrhages generally occur in primary source, or secondary bleeding at the periphery of the
the deep regions, including basal ganglia and thalamus. The most hemorrhage.
common sites for hypertensive hemorrhage are listed in Box 91.1.
ICH caused by amyloid angiopathy tends to be lobar in nature Anatomy and Physiology
and to occur more commonly in older adults.
Other factors leading to ICH include underlying vascular mal- Blood is supplied to the brain by the anterior and posterior cir-
formations (ie, arteriovenous malformations (AVMs) and aneu- culations. The anterior circulation originates from the carotid
rysms), drug intoxication (particularly sympathomimetics, such system and perfuses 80% of the brain, including the optic nerve,
as cocaine), malignant hypertension, saccular aneurysms, blood retina, and frontoparietal and anterior-temporal lobes. The first
dyscrasias, venous sinus thrombosis, hemorrhagic transformation branch off the internal carotid artery is the ophthalmic artery,
of an ischemic stroke, moyamoya disease, and tumors). High-risk which supplies the optic nerve and retina. As a result, the sudden
features for such secondary forms of ICH include lobar location, onset of painless monocular blindness (amaurosis fugax) identi-
presence of intraventricular blood, and younger age. fies the stroke as involving the anterior circulation (specifically the
ipsilateral carotid artery) at or below the level of the ophthalmic
Pathophysiology artery. The internal carotid arteries terminate by branching into
the anterior and middle cerebral arteries at the circle of Willis.
The cerebral vasculature supplies the brain with a rich flow of The anterior cerebral artery supplies the basal and medial
blood that contains the critical supply of oxygen and glucose aspects of the cerebral hemispheres and extends to the anterior
necessary for normal brain function. When a stroke occurs, there two-thirds of the parietal lobe. The middle cerebral artery feeds

Downloaded for Theo Salean (theosalean@gmail.com) at Sriwijaya University from ClinicalKey.com by Elsevier on January 17, 2020.
For personal use only. No other uses without permission. Copyright ©2020. Elsevier Inc. All rights reserved.
CH APTER 91  Stroke 1243

the lenticulostriate branches that supply the putamen, part of the the body contralateral to the side of the lesion and usually are
anterior limb of the internal capsule, the lentiform nucleus, and worse in the arm and face than the leg. Such disturbances may
the external capsule. Main cortical branches of the middle cerebral involve only part of an extremity or the face but almost always are
artery supply the lateral surfaces of the cerebral cortex from the accompanied by numbness in the same region as that of the motor
anterior portion of the frontal lobe to the posterolateral occipital loss. Hemianopsia, or blindness in one-half of the visual field,
lobe. occurs ipsilateral to the lesion. Agnosia, or the inability to recog-
Although the posterior circulation is smaller and usually sup- nize previously known subjects, is common, and aphasia may be
plies only 20% of the brain, it supplies the brainstem (which is present if the lesion occurs in the dominant hemisphere. Patients
critical for normal consciousness, movement, and sensation), cer- often have a gaze preference toward the affected hemisphere
ebellum, thalamus, auditory and vestibular centers of the ear, because of disruption of the cortical lateral gaze centers.
medial temporal lobe, and visual occipital cortex. The posterior Aphasia, a disorder of language in which the patient articulates
circulation is derived from the two vertebral arteries that ascend clearly but uses language inappropriately or understands it poorly,
through the transverse processes of the cervical vertebrae. The also is common in dominant-hemisphere stroke. Aphasia may be
vertebral arteries enter the cranium through the foramen magnum expressive, receptive, or a combination of both. Wernicke’s aphasia
and supply the cerebellum by the posterior inferior cerebellar occurs when the patient is unable to process sensory input, such
arteries. They join to form the basilar artery, which branches to as speech, and thus fails to understand verbal communication
form the posterior cerebral arteries. Some variants exist, impor- (receptive aphasia). Broca’s aphasia refers to the inability to com-
tantly, the fetal origin posterior cerebral artery, which is where municate verbally in an effective way, even though understanding
the posterior cerebral artery is actually fed by the anterior may be intact (expressive aphasia). Aphasia should be distin-
circulation. guished from dysarthria, which is a motor deficit of the mouth
The extent of injury in either an anterior or a posterior stroke and speech muscles; the dysarthric patient articulates poorly but
depends on both the vessel involved and the presence of collateral understands words and word choices. Aphasia is important to
blood flow distal to the vessel occlusion. A patient with excellent recognize because it usually localizes a lesion to the dominant
collateral blood flow from the contralateral hemisphere may have (usually left) cerebral cortex in the middle cerebral artery distri-
minimal clinical deficits despite a complete carotid occlusion. By bution. Aphasia and dysphasia are terms that are used interchange-
contrast, a patient with poor collateral flow may have hemiplegia ably but must be distinguished from dysphagia, which is difficulty
with the same lesion. in swallowing.
Pathology in the vertebrobasilar system (ie, posterior circula-
CLINICAL FEATURES tion strokes) can cause the widest variety of symptoms and as a
result may be the most difficult to diagnose. The symptoms reflect
Ischemic Stroke CN deficits, cerebellar involvement, and involvement of neurosen-
sory tracts. The brainstem also contains the reticular activating
The signs and symptoms of an ischemic stroke may appear sud- system, which is responsible for mediating consciousness, and the
denly and without warning or may have a stuttering, insidious emesis centers. Unlike those with anterior circulation strokes,
onset. Disruption of the flow to one of the major vascular limbs patients with posterior circulation stroke can have loss of con-
of the cerebral circulation will result in physiologic disruption to sciousness and frequently have nausea and vomiting. The poste-
the anatomic area of the brain supplied by that blood vessel. rior cerebral artery supplies portions of the parietal and occipital
Ischemic strokes can be classified as anterior or posterior circula- lobes, so vision and thought processing are impaired. Visual
tion strokes, depending on the vasculature involved. The presence agnosia, the inability to recognize seen objects, may be a feature,
of neurologic deficits is highly dependent on collateral flow. In as may alexia, the inability to understand the written word. A third
addition to the vascular supply involved, ischemic strokes can be nerve palsy may occur, and the patient may experience homony-
further described by the temporal presentation of their neurologic mous hemianopsia. One of the more curious facets of this syn-
deficits. drome is that the patient may be unaware of any visual problem
A “stroke in evolution” is one in which focal neurologic deficits (visual neglect). Vertigo, syncope, diplopia, visual field defects,
worsen over the course of minutes or hours. Approximately weakness, paralysis, dysarthria, dysphagia, spasticity, ataxia, or
20% of anterior circulation strokes and 40% of posterior circula- nystagmus may be associated with vertebrobasilar artery insuffi-
tion strokes will show evidence of progression. Anterior circula- ciency. Posterior circulation strokes also demonstrate crossed defi-
tion strokes may progress within the first 24 hours, whereas cits, such as motor deficits on one side of the body and sensory
posterior strokes may progress for up to 3 days. Propagation of loss on the other. In anterior circulation strokes, by contrast,
thrombus is postulated as a likely mechanism for progression. abnormalities are always limited to one side of the body.
With anterior circulation strokes (involving variously and primar- A focused neurologic examination should assess level of con-
ily the carotid, anterior, and middle cerebral arteries), the clinical sciousness, speech, CN function, motor and sensory function, and
presentation rarely includes complete loss of consciousness unless cerebellar function. Level of consciousness and fluency of speech
the lesion occurs in the previously unaffected hemisphere of a can be rapidly assessed in a dialogue with the patient to determine
patient who has experienced a previous contralateral stroke. the presence of dysarthria or aphasia. The head should be
Occlusions in the anterior cerebral artery mainly affect frontal evaluated for signs of trauma. Pupillary size and reactivity and
lobe function. The patient has altered mentation coupled with extraocular movements provide important information about
impaired judgment and insight, as well as the presence of primi- brainstem function, particularly CN III through CN VI; an abnor-
tive grasp and suck reflexes on physical examination. Bowel and mal third nerve function may be the first sign of tentorial hernia-
bladder incontinence may be features of anterior cerebral artery tion. Gaze preference suggests brainstem or cortical involvement.
stroke. Paralysis and hypesthesia of the lower limb opposite the Central facial nerve weakness from a stroke should be distin-
side of the lesion are characteristic. Leg weakness is more pro- guished from the peripheral causes of CN VII weakness. With a
nounced than arm weakness in anterior cerebral distribution peripheral lesion, the patient is unable to wrinkle the forehead.
stroke. Apraxia or clumsiness in the patient’s gait also may be Assessment of facial sensation, eyebrow elevation and squinting,
noted. smiling symmetry, gross auditory acuity, gag reflex, shoulder
Marked motor and sensory disturbances are the hallmarks of elevation, sternocleidomastoid strength, and tongue protrusion
occlusion of the middle cerebral artery. They occur on the side of complete the CN evaluation.

Downloaded for Theo Salean (theosalean@gmail.com) at Sriwijaya University from ClinicalKey.com by Elsevier on January 17, 2020.
For personal use only. No other uses without permission. Copyright ©2020. Elsevier Inc. All rights reserved.
1244 PART III  Medicine and Surgery  |  SECTION Seven  Neurology

Motor and sensory testing is performed next. Muscle tone can Hemorrhagic Stroke
be assessed by moving a relaxed limb. Proximal and distal muscle
group strength is assessed against resistance. Pronator drift of the The classic presentation of ICH is the sudden onset of headache,
arm is a sensitive sign of motor weakness and can be tested simul- vomiting, severely elevated BP, and focal neurologic deficits that
taneously by having the patient sit with eyes closed and arms progress over minutes. Similar to ischemic stroke, ICH is often
outstretched, with palms toward the ceiling, for 10 seconds. Asym- associated with a motor and sensory deficit contralateral to the
metrical sensation to pain and light touch may be subtle and brain lesion. Almost 40% of patients will demonstrate significant
difficult to detect. Double simultaneous extinction evaluation growth in hemorrhage volume within the first few hours.
tests for sensory neglect and can be easily performed by simulta- Although headache, vomiting, and coma are common, many
neously touching the right and left limbs. The patient may feel patients do not have these findings, and the clinical presentation
both the right and left sides being touched individually but may can be identical to that of patients with ischemic stroke; the two
not discern touch on one side when both are touched simultane- cannot be reliably differentiated in the absence of neuroimaging.
ously. Similarly, the ability to discern a number gently scratched Ongoing assessment of airway and mental status is of para-
on a forearm, graphesthesia, is another easily tested cortical pari- mount importance in patients with ICH because precipitous dete-
etal lobe function. These tests can help differentiate a pure motor rioration is always a possibility. Emergency airway management
deficit of a lacunar stroke from a sensorimotor middle cerebral requires careful judgment: On the one hand, airway control can
artery deficit. prevent aspiration, hypoxia, and hypercarbia; on the other, seda-
Cerebellar testing and the assessment of reflexes and gait com- tion and paralysis can make it difficult to follow the neurologic
plete the examination. Finger-to-nose and heel-to-shin evalua- examination, which can help monitor for hemorrhage expansion,
tions are important tests of cerebellar functions. Asymmetry of elevated intracranial pressure (ICP), seizure activity, and brain-
the deep tendon reflexes or unilateral Babinski’s sign may be an stem herniation.
early finding of corticospinal tract dysfunction. Gait testing is As with ischemic stroke, a careful neurologic examination is
commonly omitted yet is an informative part of the neurologic important in localizing the region and extent of injury. Baseline
examination when it can be safely performed. Observing routine NIHSS and Glasgow Coma Scale scores can be used to assess
ambulation and heel-to-toe walking can assess for subtle ataxia, stroke severity, although the Glasgow Coma Score (GCS) may be
weakness, or focal cerebellar lesions. more practical to follow for neurologic deterioration (Table 91.2).
Several prehospital stroke scales have been created to assist In addition, serial examinations can detect early changes that may
emergency medical service (EMS) personnel with the rapid assess- suggest ongoing bleeding during the acute phase. The ICH score
ment of potential stroke patients. Many of these prehospital stroke can predict mortality (Table 91.3).
scales have been prospectively validated for their accuracy in Poor prognostic indicators for patients with ICH include a
stroke detection. Two of the more commonly used scales include decreased level of consciousness on arrival, intraventricular hem-
the Cincinnati Prehospital Stroke Scale (Fig. 91.1) and Los Angeles orrhage, and large ICH volume, all of which can be assessed in
Prehospital Stroke Screen (Fig. 91.2). the emergency department (ED) (Fig. 91.3).
The National Institutes of Health Stroke Scale (NIHSS) is a
useful and rapid tool for quantifying neurologic deficit in patients DIFFERENTIAL DIAGNOSIS
with stroke and can be used in determining treatment options
(Table 91.1). NIHSS scores have been shown to be reproducible Ischemic Stroke
and valid and to correlate well with the amount of infarcted tissue
on computed tomography (CT) scan. The baseline NIHSS score Extra-axial collections of blood secondary to trauma can mimic
can identify patients who are appropriate candidates for fibrino- stroke. An epidural or subdural hematoma can cause an altered
lytic therapy, as well as those at increased risk for hemorrhage, mental status, focal neurologic signs, and rapid progression to
although it is possible for patients to have disabling strokes with coma. Elders, who represent the age group at highest risk for
an NIHSS of zero (severe truncal ataxia). In addition, it has been stroke, can be victims of recurrent falls that lead to chronic sub-
used as a prognostic tool to predict outcome and is currently dural hematomas. Carotid dissection may occur after neck trauma
being used by some stroke centers to stratify patients for entry or sudden hyperextension and may be associated with focal neu-
into treatment trials. rologic signs and symptoms, as with an aortic dissection that
extends into the carotid arteries.
Other structural lesions that may cause focal neurologic signs
include brain tumors and abscesses. Air embolism should be sus-
pected in the setting of marked atmospheric pressure changes,
such as in scuba diving or during medical procedures or injuries
that may allow air into the vascular system. Seizures, altered
Cincinnati Prehospital Stroke Scale mental status, and focal neurologic findings also may be manifes-
tations of air embolism.
Facial Droop Metabolic abnormalities also can mimic stroke syndromes.
Normal: Both sides of face move equally
Hypoglycemia often is responsible for an altered mental status and
Abnormal: One side of face does not move at all
is a well-known cause of sustained focal neurologic symptoms
Arm Drift that can persist for several days. Wernicke’s encephalopathy causes
Normal: Both arms move equally or not at all ophthalmoplegia, ataxia, and confusion that can be mistaken for
Abnormal: One arm drifts compared to the other signs of cerebellar infarction.
Speech Migraine may present with focal neurologic findings, with or
Normal: Patient uses correct words with no slurring without headache. A seizure followed by Todd’s postictal paralysis
Abnormal: Slurred or inappropriate words or mute may mimic stroke. Bell’s palsy, labyrinthitis, vestibular neuronitis,
peripheral nerve palsy, and demyelinating diseases may all mimic
Fig. 91.1.  Cincinnati Prehospital Stroke Scale. (Adapted from Kothari stroke. Ménière’s disease may be difficult to distinguish from a
RU, Pancioli A, Liu T, et al: Cincinnati Prehospital Stroke Scale: reproduc- posterior circulation stroke or TIA. Dizziness, vertigo, hearing
ibility and validity. Ann Emerg Med 33[4]:373-378, 1999.) loss, and tinnitus in Ménière’s disease are common, whereas

Downloaded for Theo Salean (theosalean@gmail.com) at Sriwijaya University from ClinicalKey.com by Elsevier on January 17, 2020.
For personal use only. No other uses without permission. Copyright ©2020. Elsevier Inc. All rights reserved.
CH APTER 91  Stroke 1245

Los Angeles Patient name:


Prehospital Rater name:
Stroke Scale (LAPSS) Date:

Screening criteria Yes No

4. Age over 45 years

5. No prior history of seizure disorder

6. New onset of neurologic symptoms in last 24 hours

7. Patient was ambulatory at baseline (prior to event)

8. Blood glucose between 60 and 400

9. Exam: Look for obvious asymmetry

Normal Right Left


Facial smile / grimace: Droop Droop

Grip: Weak grip Weak grip


No grip No grip

Arm weakness: Drifts down Drifts down


Falls rapidly Falls rapidly

Based on exam, patient has only unilateral (and not bilateral) weakness: Yes No

10. If yes (or unknown) to all items above LAPSS screening criteria met: Yes No

11. If LAPSS criteria for stroke met, call receiving hospital with “CODE STROKE,” if not
then return to the appropriate treatment protocol. (Note: The patient may still be
experiencing a stroke if even if LAPSS criteria are not met.)

Provided by the internet stroke center — www.strokecenter.org

Fig. 91.2.  Los Angeles Prehospital Stroke Screen. (Adapted from Kidwell CS, Starkman S, Eckstein M,
et al: Identifying stroke in the field: prospective validation of the Los Angeles Prehospital Stroke Screen
[LAPSS]. Stroke 31:71-76, 2000.)

difficulties with vision or speech or other focal symptoms are nodosa, lupus, and other types of vasculitis may cause stroke
uncommon. syndromes.
Like stroke, giant cell arteritis is a disease of older adults. It may Cerebral venous sinus thrombosis (CVST) is another cause of
cause severe headache, visual disturbances, and, rarely, aphasia focal neurologic symptoms that most commonly affects the supe-
and hemiparesis. Other symptoms include intermittent fever, rior sagittal sinus and lateral sinuses (see Chapter 93).8 The diag-
malaise, jaw claudication, morning stiffness, and myalgias. The nosis of CVST can be difficult because of the nonspecific nature
diagnosis should be suspected in patients with a very high eryth- of symptoms, as well as the variable time frame of symptom onset
rocyte sedimentation rate (ESR) and is confirmed by temporal (from hours to a few weeks). Patients may have generalized head-
artery biopsy. Collagen vascular diseases such as polyarteritis aches, nausea, vomiting, paresis, visual disturbances, depressed

Downloaded for Theo Salean (theosalean@gmail.com) at Sriwijaya University from ClinicalKey.com by Elsevier on January 17, 2020.
For personal use only. No other uses without permission. Copyright ©2020. Elsevier Inc. All rights reserved.
1246 PART III  Medicine and Surgery  |  SECTION Seven  Neurology

TABLE 91.1 

National Institutes of Health Stroke Scale Scoring Form


ITEM SCORING DEFINITIONS SCORE
1a. Level of consciousness (LOC) 0 = Alert and responsive
1 = Arousable to minor stimulation
2 = Arousable only to painful stimulation
3 = Reflex responses or unarousable
1b. LOC-related questions: Ask patient’s age 0 = Both correct
and month. Must be exact. 1 = One correct (or dysarthria, intubated, foreign language)
2 = Neither correct
1c. Commands: Open and close eyes, grip and 0 = Both correct (acceptable if impaired by weakness)
release nonparetic hand. (Other one-step 1 = One correct
commands or mimic also acceptable.) 2 = Neither correct
2. Best gaze: Horizontal EOM by voluntary or 0 = Normal
doll’s eye maneuver. 1 = Partial gaze palsy; abnormal gaze in one or both eyes
2 = Forced eye deviation or total paresis that cannot be overcome by doll’s eye maneuver
3. Visual field: Use visual threat if necessary. 0 = No visual loss
If monocular, score field of good eye. 1 = Partial hemianopsia, quadrantanopia, extinction
2 = Complete hemianopsia
3 = Bilateral hemianopsia or blindness
4. Facial palsy: If patient is stuporous, check 0 = Normal
symmetry of grimace to pain. 1 = Minor paralysis, flat NLF, asymmetrical smile
2 = Partial paralysis (lower face = UMN lesion)
3 = Complete paralysis (upper and lower face)
5. Motor arm: Arms outstretched 90 degrees 0 = No drift for 10 seconds L or R
(sitting) or 45 degrees (supine) for 10 1 = Drift but does not hit bed
seconds. Encourage best effort. Indicate 2 = Some antigravity effort, but cannot sustain
paretic limb in score box. 3 = No antigravity effort, but even minimal movement counts
4 = No movement at all
X = Unable to assess owing to amputation, fusion, fracture, and so on
6. Motor leg: Raise leg to 30 degrees (from 0 = No drift for 5 seconds L or R
supine) for 5 seconds. Indicate paretic limb 1 = Drift but does not hit bed
in score box. 2 = Some antigravity effort, but cannot sustain
3 = No antigravity effort, but even minimal movement counts
4 = No movement at all
X = Unable to assess owing to amputation, fusion, fracture, and so on
7. Limb ataxia: Check finger-nose-finger, 0 = No ataxia (or aphasic, hemiplegic) L or R
heel-shin position sense; and score only if 1 = Ataxia in upper or lower extremity
out of proportion to paralysis. 2 = Ataxia in upper and lower extremity
X = Unable to assess owing to amputation, fusion, fracture, and so on
8. Sensory: Use safety pin. Check grimace or 0 = Normal
withdrawal if patient is stuporous. Score 1 = Mild-moderate unilateral loss but patient aware of touch (or aphasic, confused)
only stroke-related losses. 2 = Total loss, patient unaware of touch; coma, bilateral loss
9. Best language: Describe cookie jar picture, 0 = Normal
name objects, read sentences. May use 1 = Mild-moderate aphasia (speech difficult to understand but partly comprehensible)
repeating, writing, stereognosis. 2 = Severe aphasia (almost no information exchanged)
3 = Mute, global aphasia, coma; no one-step commands
10. Dysarthria: Read list of words. 0 = Normal
1 = Mild-moderate; slurred but intelligible
2 = Severe; unintelligible or mute
X = Intubation or mechanical barrier
11. Extinction or neglect: Simultaneously check 0 = Normal, none detected (visual loss alone)
bilateral visual fields and auditory 1 = Neglects or extinguishes to double simultaneous stimulation in any modality (visual,
sensation, touch in both hands and auditory, sensation, spatial, body parts)
recognition of body parts looking for 2 = Profound neglect in more than one modality
extinction or neglect.
Android Free App: https://play.google.com/store/apps/details?id=com.myprograms.nihss
Apple Free App: https://itunes.apple.com/us/app/nih-stroke-scale-from-statcoder/id408788598?mt=8
Online NIHSS Calculator: www.mdcalc.com/nih-stroke-scale-score-nihss/
EOM, Extraocular movement; L, left; LOC, level of consciousness; NLF, nasolabial fold; R, right; UMN, upper motor neuron.
Modified from Massachusetts General Hospital Stroke Service. NIH stroke scale materials. Scoring form. Available at www2.massgeneral.org/stopstroke/pdfs/scoring
_form.pdf.

Downloaded for Theo Salean (theosalean@gmail.com) at Sriwijaya University from ClinicalKey.com by Elsevier on January 17, 2020.
For personal use only. No other uses without permission. Copyright ©2020. Elsevier Inc. All rights reserved.
CH APTER 91  Stroke 1247

TABLE 91.2  TABLE 91.3 

Glasgow Coma Scale Score* Intracerebral Hemorrhage Score Predicting Mortality


EYE OPENING VERBAL RESPONSE MOTOR RESPONSE After Acute Intracerebral Hemorrhage
(E) (V) (M) FEATURE POINTS
4 = Spontaneous 5 = Normal conversation 6 = Normal GLASGOW COMA SCALE SCORE
3 = To voice 4 = Disoriented 5 = Localizes to pain
2 = To pain conversation 4 = Withdraws to pain 3 to 4 2
1 = None 3 = Words, but not 3 = Decorticate posture 5 to 12 1
coherent 2 = Decerebrate posture
2 = No words; only 1 = None 13 to 15 0
sounds INTRACEREBRAL HEMORRHAGE VOLUME
1 = None
>30 mL 1
*Total score = E + V + M
Shoestring Graphics: Glasgow coma score. ≤30 mL 0
Available at www.ssgfx.com/CP2020/medtech/glossary/glasgow.htm. INTRAVENTRICULAR HEMORRHAGE
(INTRAVENTRICULAR BLOOD)
Present 1
Absent 0
INTRACEREBRAL HEMORRHAGE LOCATION
Infratentorial 1
Supratentorial 0
AGE
≥80 years 1
A
<80 years 0
B 30-DAY MORTALITIES FOR TOTAL INTRACEREBRAL
HEMORRHAGE SCORES
0 = 0%
1 = 13%
2 = 26%
3 = 72%
4 = 97%
5 = 100%

Fig. 91.3.  The computed tomography (CT) slice with the largest area of
6 = Estimated to be 100%; no patients in the study fell into this
hemorrhage is identified. The largest diameter of the hemorrhage on this category
slice is measured in centimeters (line A). The largest diameter 90 degrees Adapted from Hemphill JC, et al: The ICH Score. Stroke 32:891-897, 2001.
to A on the same slice is measured (line B). C is the approximate number
of 10-mm slices on which the intracerebral hemorrhage (ICH) was seen.
(Many centers use 5-mm slices, in which case an adjustment can be made
by dividing by 2.) The volume of the hemorrhage = A × B × C ÷ 2 (ABC/2).
and vomiting, although focal neurologic signs are less common
in these entities. With hypertensive encephalopathy, patients
usually exhibit marked elevation in BP and other evidence of
level of consciousness, seizures, or even symptoms generally end-organ injury, such as proteinuria, cardiomegaly, papilledema,
ascribed to psychiatric disorders (such as, depression). Depending and malignant hypertensive retinopathy. These patients usually
on the location of the thrombus, physical examination of the improve significantly with treatment of their hypertension. The
patient may reveal papilledema, proptosis, or palsies of CNs III, posterior reversible encephalopathy syndrome is an important
IV, and VI, as well as other focal neurologic signs and symptoms. subset of hypertensive encephalopathy presentations and has
Risk factors for CVST include trauma, infectious processes, hyper- characteristic CT or magnetic resonance imaging (MRI) changes.
coagulable states, low-flow states, compression of the venous Once ICH is appreciated on neuroimaging, it can be difficult
sinus, dehydration, various drugs (such as, androgens, “ecstasy,” to determine the underlying cause. Primary ICH typically mani-
and oral contraceptives), and pregnancy or the postpartum state. fests as a parenchymal hematoma with new onset neurologic
symptoms. Patients with hemorrhagic transformation of an isch-
Hemorrhagic Stroke emic stroke may have recurrence or worsening of previously
established deficits. Patients with known underlying cancer, or
The differential diagnosis for ICH is similar to that for ischemic perihematomal edema out of proportion to the hemorrhage,
stroke; considerations include migraine, seizure, tumor, abscess, should be considered for hemorrhage into a metastasis or primary
hypertensive encephalopathy, and trauma. Hypertensive encepha- tumor. Finally, patients with known underlying venous thrombo-
lopathy and migraine also can manifest with headache, nausea, embolic risk factors may have underlying CVST.

Downloaded for Theo Salean (theosalean@gmail.com) at Sriwijaya University from ClinicalKey.com by Elsevier on January 17, 2020.
For personal use only. No other uses without permission. Copyright ©2020. Elsevier Inc. All rights reserved.
1248 PART III  Medicine and Surgery  |  SECTION Seven  Neurology

DIAGNOSTIC TESTING

Ischemic Stroke
Although clinical data can help establish the diagnosis, cause, and
location of the stroke, confirmatory diagnostic tests are often
required to establish the final cause or to eliminate other causes
for the deficits. The immediate evaluation includes cranial
imaging, an electrocardiogram (ECG) and hematologic testing,
particularly blood glucose determination.
An emergent noncontrast cranial CT is the standard initial
imaging technique for evaluating a patient with a potential stroke.
It can quickly differentiate an ischemic stroke from ICH and other
mass lesions. This information is crucial to the subsequent thera-
peutic decisions that will be rapidly made. A CT scan can identify
almost all parenchymal hemorrhages larger than 1 cm in diameter
and it has a high sensitivity for the detection of SAH.9 In a major-
ity of ischemic strokes, gross signs of infarction will not appear
on routine CT scans for at least 6 to 12 hours, depending on the
size of the infarct. However, subtle, early ischemic changes have A
been noted in up to 67% of noncontrast CT scans within the first
3 hours. These early ischemic changes include the hyperdense
artery sign (acute thrombus in a vessel), sulcal effacement, loss of
the insular ribbon, loss of gray-white interface, mass effect, and
acute hypodensity (Fig. 91.4). In addition, CTA can be used to
identify the presence of intravascular thrombosis, vasculature dis-
section, or stenosis. In cases in which arterial dissection is sus-
pected, imaging with MRA or CTA is indicated.10
The clinical importance of early ischemic CT findings with
regard to fibrinolytic therapy within 3 hours of symptom onset is
questionable, because the ability of treating physicians to repro-
ducibly identify these findings is poor and their clinical signifi-
cance is questionable. Only acute hypodensity and mass effect
have been shown to be associated with an increased risk of
ICH after fibrinolysis (over that in treated patients without
these findings). However, these findings do not exclude patients
from fibrinolytic therapy, which is associated with an improved
neurologic outcome. Patients with a hyperdense artery sign and
acute hypodensity of one-third of the middle cerebral artery
distribution tend to have a poorer prognosis; however, their
outcomes are still better with tPA treatment than without such B
treatment.
MRI can visualize ischemic infarcts earlier and identify acute Fig. 91.4.  A, Computed tomography (CT) scan taken 2 hours and 50
posterior circulation strokes more accurately than CT, and it may minutes after a large right middle cerebral artery occlusion. There are
be as effective as CT in identifying ICH.10 However, availability, subtle, ultra-early ischemic changes, including loss of the gray-white
interface (arrows) and subtle evidence of sulcal effacement. B, CT scan
difficulty in accessing critically ill patients, and scan time limit the
of same patient approximately 8 hours after symptom onset shows acute
use of MRI in acute stroke. Advances in MRA technology have hypodensity (arrows) and more prominent sulcal effacement.
allowed a noninvasive method of demonstrating large-vessel
occlusions of the anterior and posterior circulation, although
small intracranial vascular occlusions may not be readily appar-
ent. With the improvements in MRI and MRA speed and resolu-
tion, some stroke centers are replacing CT protocols with limited
“stroke protocol” MRI or MRA as the initial imaging modality of brain. In addition, CTA and perfusion CT can differentiate revers-
choice. The choice of initial cranial imaging modality is highly ible from irreversible ischemic insults.
dependent on the speed with which these scans can be performed An ECG is indicated in all patients with acute ischemic stroke,
and interpreted at each individual center. because atrial fibrillation and acute myocardial infarction are
Diffusion-weighted imaging (DWI) and perfusion-weighted associated with up to 60% of all cardioembolic strokes. The hema-
imaging (PWI) are MRI techniques that take minutes to perform tologic evaluation includes a complete blood count with platelet
and may allow differentiation between reversible and irreversible count, prothrombin time (including international normalized
neuronal injury. Other potential imaging modalities include CTA ratio [INR]), partial thromboplastin time, cardiac enzymes, and
and perfusion scans. In CTA, a CT scan is enhanced by an intra- serum glucose measurement. Elevated blood viscosity, even when
venous (IV) contrast agent to better define the vasculature of the hematocrit levels are not frankly polycythemic, can affect blood
brain. Areas of vascular stenosis and occlusion can be visualized flow and prognosis. A platelet count can identify thrombocytosis
with this technique. This information can then be used by inter- or thrombocytopenia, which may precipitate a thrombosis or
ventionalists to determine whether a lesion is amenable to endo- hemorrhage. Coagulation studies are especially helpful to guide
vascular thrombectomy. Also requiring IV contrast, perfusion CT management for patients in whom anticoagulation is being con-
scans can reveal perfusion deficits within different regions of the sidered or for patients with a hemorrhagic stroke.

Downloaded for Theo Salean (theosalean@gmail.com) at Sriwijaya University from ClinicalKey.com by Elsevier on January 17, 2020.
For personal use only. No other uses without permission. Copyright ©2020. Elsevier Inc. All rights reserved.
CH APTER 91  Stroke 1249

Other ancillary diagnostic tests to consider include an echo- TABLE 91.4 


cardiogram, carotid duplex scan, and angiogram. Some centers
are performing these studies as part of a TIA observation unit National Institute of Neurological Disorders and
protocol to exclude a patent foramen ovale or valvular vegetation Stroke Recommended Stroke Evaluation Targets for
in those patients in whom a cardioembolic stroke is suspected. An
echocardiogram should also be performed in patients with no Potential Thrombolytic Candidates
obvious cause for their stroke.11 Finally, conventional angiography MANAGEMENT COMPONENT TARGET TIME FRAME
can demonstrate stenosis or occlusion of both large and small
blood vessels of the head and neck. It can detect subtle abnormali- Door to doctor 10 minutes
ties, such as with dissection, that may not be demonstrated with Door to CT completion 25 minutes
noninvasive imaging techniques.
Door to CT scan reading 45 minutes
Door to treatment 60 minutes
Transient Ischemic Attack
Access to neurologic expertise* 15 minutes
Patients with new-onset TIAs should receive an expedited evalu-
ation and treatment owing to the substantial short-term risk of Access to neurosurgical expertise* 2 hours
stroke and other adverse events.12 Emergency neuroimaging, vas- *By phone or in person.
cular imaging (such as, with a carotid Doppler study, MRA, or CT, computed tomography.
CTA), electrocardiography, and basic blood tests should be per-
formed. A medically or surgically treatable cause for TIAs (eg,
high-grade carotid stenosis, mural thrombus) should be sought,
which would require in-hospital treatment such as anticoagula- unresponsive on presentation, their ability to communicate may
tion, stenting, or carotid endarterectomy. be altered by dysphasia. After an ischemic stroke, patients usually
can maintain their airway unless the brainstem is affected or sig-
Hemorrhagic Stroke nificant cerebral edema is compressing the opposite hemisphere.
Patients with intact protective airway reflexes should receive
The hematologic evaluation for the patient with hemorrhagic oxygen if they are hypoxic (oxygen saturation less than 95%), and
stroke should be performed in the same manner as for the patient a monitor and IV line should be established. Routine oxygen
with ischemic stroke. Particular attention should be directed to supplementation of normoxic stroke patients should be avoided.13
uncovering the presence of a coagulopathy. A drug screen should Overhydration should be avoided to prevent cerebral edema.
be obtained to evaluate for use of sympathomimetics if substance By contrast, dehydration may lead to decreased cerebral perfusion,
abuse is suspected. Increased sympathetic outflow secondary to and saline infusion should be given if dehydration is suspected.
the hemorrhage may lead to an increase in dysrhythmias. Dys- Dextrose-containing solutions should be avoided in normoglyce-
rhythmias also may signal impending brainstem compression mic patients suspected of having had a stroke because elevated
from an expanding hemorrhage. blood glucose levels may worsen an ischemic deficit. Out-of-
As in ischemic stroke, the cranial CT scan is the diagnostic test hospital personnel should attempt to rapidly ascertain the patient’s
of choice to evaluate for an ICH. The noncontrast CT scan will blood sugar; if this is not possible, glucose should be given when
reliably diagnose patients with clinically relevant acute ICH. hypoglycemia is strongly suspected with an understanding that
Hemorrhages that are several days old may not be as apparent as hyperglycemia may be neurotoxic. Electrocardiographic monitor-
acute hemorrhages and appear as isodense regions. ing is recommended to identify life-threatening arrhythmias and
Also, as with ischemic stroke, advanced neuroimaging modali- atrial fibrillation.
ties are gaining favor in ICH. CTA produces high-quality images The circumstances surrounding the stroke as well as concomi-
of the larger arterial vessels and can help exclude secondary causes, tant medical conditions should be ascertained. A key part of the
such as aneurysm, AVM, or fistula. Some patients with primary initial information on stroke patients is the prehospital providers’
ICH show contrast extravasation on CTA, and such patients are documentation of the exact time the patient was last seen to be
at particularly high risk of ongoing bleeding and hematoma neurologically normal and the level of neurologic functioning.
expansion. A venous phase can be added to this study (computed This is especially important because reversible defects may com-
tomography venography [CTV]) to evaluate for CVST. A MRI can pletely resolve by the time the patient has arrived at the hospital.
help detect underlying lesions (such as, tumor) and may offer The level of consciousness, gross focal motor deficits, difficulty
better resolution for evaluating perihematomal edema. When with speech, clumsiness, facial asymmetry, and any other focal
available, MRA and magnetic resonance venography (MRV) can deficits should be noted. Prehospital stroke scales assist in identi-
be used in place of CTA and CTV. fying patients who have had a stroke and who are potential can-
didates for fibrinolytic therapy. Early recognition, notification,
MANAGEMENT and transport by EMS are associated with delivery of fibrinolytic
treatment and improved patient outcomes.14
Ischemic Stroke In the ED setting, the vital signs should be reassessed on an
ongoing basis because patients may rapidly deteriorate even with
With a focus on rapid recognition, evaluation, and treatment of subacute stroke. Some stroke patients are found at home 1 or 2
stroke, many hospitals have streamlined care to meet recom- days after the event has occurred and may have concomitant ill-
mended time goals (Table 91.4). This has led to the development nesses, such as aspiration pneumonia, dehydration, hypothermia,
of stroke protocols, critical pathways, and acute interventional rhabdomyolysis, or myocardial ischemia. Fever necessitates an
stroke teams that may even be deployed in the field before the evaluation to identify sources of infection, followed by prompt
patient arrives at the ED. institution of treatment. Even minor degrees of hyperthermia
In the prehospital setting, the focus should be on ensuring have been associated with increased neurologic injury. Oral medi-
central nervous system (CNS) oxygenation and perfusion, rapid cations (and food) should be withheld until some form of swal-
identification, early hospital notification, and rapid transport. lowing assessment has been performed, given the risk of aspiration
Although it is unusual for patients with ischemic stroke to be in patients with an acute stroke.

Downloaded for Theo Salean (theosalean@gmail.com) at Sriwijaya University from ClinicalKey.com by Elsevier on January 17, 2020.
For personal use only. No other uses without permission. Copyright ©2020. Elsevier Inc. All rights reserved.
1250 PART III  Medicine and Surgery  |  SECTION Seven  Neurology

Blood Pressure Management nifedipine or sublingual nitroglycerin are not recommended,


because either agent can produce a precipitous drop in BP.
The management of BP in patients with acute ischemic stroke and If fibrinolytic therapy is planned, stringent control of BP is
TIA is controversial because of limited data. Current guidelines indicated to reduce the potential for intracranial hemorrhage after
for the management of hypertension in patients with acute isch- the thrombolytic is administered (see Box 91.2).
emic stroke recommend that antihypertensive treatment be Thrombolytic therapy is not recommended for patients whose
reserved for those with markedly elevated BPs, unless fibrinolytic systolic pressure is consistently higher than 185 mm Hg or whose
therapy is planned or specific medical indications are present. diastolic pressure is 110 mm Hg at the time of treatment. Simple
These medical indications include acute myocardial infarction, measures can be used to try lowering BP below this level. Recom-
aortic dissection, hypertensive encephalopathy, and severe left mended approaches include the use of IV labetalol 10 to 20 mg
ventricular heart failure. or continuous nicardipine. Once thrombolytic therapy has been
Oral or parenteral agents are withheld unless the patient’s sys- initiated, BP must be monitored closely and hypertension treated
tolic pressure is greater than 220 mm Hg, diastolic pressure is aggressively.
greater than 120 mm Hg, or mean arterial pressure (MAP) is Just as problematic as high BP can be, low BP can be quite
greater than 130 mm Hg (Box 91.2). If parenteral agents are used, detrimental to patients with ischemic stroke. Normally normo-
labetalol 10 to 20 mg IV push, or a calcium channel blocker (eg, tensive stroke patients with low BP or normally hypertensive
nicardipine starting at 5 mg/hour IV), is favored because of ease stroke patients with low or even low-normal BP are given a fluid
of titration and limited effect on cerebral blood vessels. Sublingual bolus to try to increase cerebral perfusion. This is especially
important in patients in a dehydrated state. If initial fluid chal-
lenge is ineffective, the patient may require vasopressor therapy
(eg, with dopamine) to gradually increase MAP and improve cere-
BOX 91.2  bral perfusion.

Emergency Antihypertensive Therapy for Acute Thrombolytic Therapy


Ischemic Stroke To date, the only IV thrombolytic agent approved by the U.S. Food
and Drug Administration (FDA) for treatment of patients with
INDICATION THAT PATIENT IS ELIGIBLE FOR TREATMENT acute ischemic stroke is the recombinant tissue plasminogen acti-
WITH INTRAVENOUS RECOMBINANT TISSUE PLASMINOGEN vator (rtPA), alteplase (Activase). Approval was initially based on
ACTIVATOR OR OTHER ACUTE REPERFUSION INTERVENTION the results of the National Institute of Neurological Disorders and
Blood Pressure Level Stroke (NINDS) trial, although subsequent analysis of other
Systolic >185 mm Hg or diastolic >110 mm Hg
studies has supported its use.15,16 There was initial concern regard-
Labetalol 10 to 20 mg IV over 1 to 2 minutes; may repeat 1 time
ing the safety of alteplase when used in community practice;
or
Nicardipine infusion, 5 mg/hr; titrate up by 2.5 mg/hr at 5- to however, a meta-analysis of non–trial-related use in community
15-minute intervals, maximum dose 15 mg/hr; when desired BP practice has demonstrated efficacy and safety similar to that
attained, reduce to 3 mg/hr reported in the NINDS trial; this was also replicated within a
Other agents (hydralazine, enalaprilat, and so on) may be considered cluster randomized controlled trial performed in Michigan.17 The
when appropriate. recommended dose for rtPA is 0.9 mg/kg IV to a maximum of
If BP does not decline and remains >185/110 mm Hg, do not 90 mg (10% of the dose given as a bolus followed by an infusion
administer rtPA. lasting 60 minutes). Although the initial recommended time
window for IV rtPA administration was 3 hours because the
MANAGEMENT OF BLOOD PRESSURE DURING AND AFTER patient was last known to be at their neurologic baseline, a sub-
TREATMENT WITH RECOMBINANT TISSUE PLASMINOGEN sequent study has demonstrated the usefulness of IV rtPA at 3 to
ACTIVATOR OR OTHER ACUTE REPERFUSION INTERVENTION 4.5 hours in a carefully selected subgroup of acute ischemic stroke
Monitor BP every 15 minutes during treatment and then for another 2 patients (Table 91.5 and Box 91.3). A larger open-label random-
hours, then every 30 minutes for 6 hours, and then every hour for 16 ized trial focusing on patients presenting with “reasonable uncer-
hours. tainty” regarding the expected benefit of rtPA found reduced
death and dependency at 6 months; a large proportion of patients
Blood Pressure Level in this study were quite old, or had old stroke (>4.5 hours), or
Systolic 180 to 230 mm Hg or diastolic 105 to 120 mm Hg both.18 These results should not influence clinical practice, because
Labetalol 10 mg IV over 1 to 2 minutes; may repeat every 10 to 20 the bulk of the population included in this large trial was mark-
minutes; maximum dose of 300 mg
edly different from the population of stroke patients who receive
or
thrombolysis within the context of current guidelines.
Labetalol 10 mg IV followed by an infusion at 2 to 8 mg/min
Systolic >230 mm Hg or diastolic 121 to 140 mm Hg The American Stroke Association recommends that rtPA be
Labetalol 10 mg IV over 1 to 2 minutes; may repeat every 10 to 20 given within 60 minutes of arrival to appropriately selected isch-
minutes; maximum dose of 300 mg emic stroke patients. Recent guidelines from the American College
or of Emergency Physicians concur but also emphasize that proper
Labetalol 10 mg IV followed by an infusion at 2 to 8 mg/min systems of care must also be in place to ensure safety and to maxi-
or mize good outcomes.19
Nicardipine infusion, 5 mg/hr; titrate up to desired effect by Studies suggests that patients with mild or rapidly resolving
increasing 2.5 mg/hr every 5 minutes to maximum of 15 mg/hr symptoms may still benefit from the use of IV rtPA, and a clinical
If BP not controlled, consider sodium nitroprusside. trial is ongoing in this area. A promising early phase trial
of tenecteplase was recently completed for patients with mild
BP, Blood pressure; IV, intravenous; rtPA, recombinant tissue plasminogen activator.
Adapted from Jauch EC, Saver JL, Adams HP Jr, et al: Guidelines for the early manage-
stroke and high-grade large-vessel stenosis. However, this strategy
ment of adults with ischemic stroke: a guideline for healthcare professionals from the cannot be recommended yet. If feasible clinicians should consider
American Heart Association/American Stroke Association. Stroke 44(3):870-947, urgent transfer of patients with mild symptoms and high grade
2013. stenosis for potential reperfusion strategies, including mechanical

Downloaded for Theo Salean (theosalean@gmail.com) at Sriwijaya University from ClinicalKey.com by Elsevier on January 17, 2020.
For personal use only. No other uses without permission. Copyright ©2020. Elsevier Inc. All rights reserved.
CH APTER 91  Stroke 1251

TABLE 91.5 

Comparison of AHA/ASA Acute Stroke Management Guidelines and Previous and New FDA Prescribing
Information for Alteplase (Activase) Treatment in Acute Ischemic Stroke
Inclusion criteria for fibrinolytic therapy
• Diagnosis of ischemic stroke causing measurable neurological deficit
• Onset of symptoms less than 3 hours before beginning treatment
AHA/ASA ACUTE STROKE OLD ALTEPLASE (ACTIVASE) PI NEW ALTEPLASE (ACTIVASE)
CRITERION MANAGEMENT GUIDELINE 2013a (UPDATED 2009) PI (FEBRUARY 2015)
Prior stroke Exclusion: prior stroke within 3 mo Contraindication: recent (within 3 mo) previous Removed entirely
stroke
Seizure at onset Relative exclusion: seizure at onset Contraindication: seizure at the onset of stroke Removed entirely
with postictal neurological
impairments
Bleeding diathesis/ Exclusion: Contraindication: known bleeding diathesis Bleeding diathesis remains a
OACs Platelet count <100 000/mm3 including but not limited to: contraindication, but all
Heparin received within 48 h, Current use of OACs (eg, warfarin sodium), laboratory values and specific
resulting in abnormally an INR >I.7, or a PT >15 s examples removed
elevated aPTT Administration of heparin within 48 h
Current use of anticoagulant with preceding the onset of stroke with an
INR >1.7 or PT >15 s elevated aPTT at presentation
Current use of direct thrombin Platelet count <100 000/mm3
inhibitors or direct factor Xa Warning for all indications: patients currently
inhibitors with elevated taking OACs
sensitive laboratory tests
ICH Exclusion: history of previous ICH Contraindication: history of ICH Contraindication removed
Warning added for recent ICH
BP Exclusion: Elevated BP (systolic Contraindication: uncontrolled hypertension at Contraindication: current severe
>85 mm Hg or diastolic the time of treatment (eg, >185 mm Hg uncontrolled hypertension
>10 mm Hg) systolic or >110 mm Hg diastolic) remains, specific BP values
removed
Warning for BP >175/110 mm Hg
remains for all alteplase
(Activase) indications
Blood glucose Exclusion: blood glucose <50 mg/dL Warning: because of the increased risk for Removed entirely
misdiagnosis of acute ischemic stroke, special
diligence is required in making this diagnosis
in patients whose blood glucose values are
≈50 or >400 mg/dL
Severe stroke Not listed Warning: patients with severe neurological deficit Removed entirely
(NIHSS score >22) at presentation; there is
an increased risk of ICH in these patients
Mild stroke Relative exclusion: only minor or Warning: safety and efficacy in patients with Removed entirely
rapidly improving stroke symptoms minor neurological deficit or with rapidly
(clearing spontaneously) improving symptoms have not been
evaluated; therefore, treatment of patients
with minor neurological deficit or with rapidly
improving symptoms is not recommended
Neuroimaging Exclusion: CT demonstrates multilobar Warning: Major early infarct sign (substantial Removed entirely
findings infarction (hypodensity >1/3 edema, mass effect, or midline shift on CT)
cerebral hemisphere)
SAH Exclusion: symptoms suggest SAH Contraindication: Suspicion of SAH on Contraindication: subarachnoid
pretreatment evaluation hemorrhage
Continued

Downloaded for Theo Salean (theosalean@gmail.com) at Sriwijaya University from ClinicalKey.com by Elsevier on January 17, 2020.
For personal use only. No other uses without permission. Copyright ©2020. Elsevier Inc. All rights reserved.
1252 PART III  Medicine and Surgery  |  SECTION Seven  Neurology

TABLE 91.5 

Comparison of AHA/ASA Acute Stroke Management Guidelines and Previous and New FDA Prescribing
Information for Alteplase (Activase) Treatment in Acute Ischemic Stroke—cont’d
AHA/ASA ACUTE STROKE OLD ALTEPLASE (ACTIVASE) PI NEW ALTEPLASE (ACTIVASE)
CRITERION MANAGEMENT GUIDELINE 2013a (UPDATED 2009) PI (FEBRUARY 2015)
Use in specific
populations
 Pregnancy Relative exclusion Warning: pregnancy No change
Category C
  Nursing mothers Not listed Not mentioned Unknown risk
 Children Inclusion: ≥18 y of age Indicated for adults Pediatric use not established
 Elderly Not listed Warning for all indications: advanced age (eg, Warning added: age >77 y was 1
>75 y) may increase risks of several interrelated baseline
characteristics associated with
an increased risk of ICH;
efficacy results suggest a
reduced but still favorable
clinical outcome
Gastrointestinal or Warning: gastrointestinal or Warning: gastrointestinal or genitourinary Warning: gastrointestinal or
genitourinary genitourinary bleeding within the bleeding within the past 21 d genitourinary bleeding
bleeding past 21 d
a
From Jauch EC, Saver JL, Adams HP Jr, et al; on behalf of the American Heart Association Stroke Council; Council on Cardiovascular Nursing; Council on Peripheral Vascular
Disease; Council on Clinical Cardiology. Guidelines for the early management of patients with acute ischemic stroke: a guideline for healthcare professionals from the American
Heart Association/American Stroke Association. Stroke 44:870–947, 2013.
AHA/ASA, American Heart Association/American Stroke Association; aPTT, activated partial thromboplastin time; BP, blood pressure; CT, computed tomography; FDA, US Food
and Drug Administration; ICH, intracerebral hemorrhage; INR, international normalized ratio; NIHSS, National Institute of Health Stroke Scale; OAC, oral anticoagulant; PI,
prescribing information; PT, prothrombin time; SAH, subarachnoid hemorrhage.
From Demaerschalk BM, Kleindorfer DO, Adeoye OM, et al; American Heart Association Stroke Council and Council on Epidemiology and Prevention. Scientific Rationale for
the Inclusion and Exclusion Criteria for Intravenous Alteplase in Acute Ischemic Stroke: A Statement for Healthcare Professionals From the American Heart Association/American
Stroke Association. Stroke 47(2):581–641, 2016.

BOX 91.3  patient will have differing probabilities of benefits and risks.
Optimal methods of communicating benefits and risks promptly
Fibrinolytic Therapy for Acute Ischemic Stroke have not been developed or widely disseminated, because
all reviewed decision aids have methodological deficiencies.22
in the 3- to 4.5-Hour Time Window Inclusion However, younger patients and milder strokes (NIHSS < 10)
and Exclusion Criteria clearly have a lower risk of SICH. It is reasonable to adjust outcome
expectations when discussing benefits and risks with older patients
INCLUSION CRITERIA with more severe strokes (including significant early CT changes).
Diagnosis of ischemic stroke causing measurable neurological deficit The risk of severe disability and death is quite high in this popula-
Onset of symptoms within 3 to 4.5 hours before beginning treatment tion with or without fibrinolysis. Prompt fibrinolysis of severe
strokes will shift the outcomes of a population of treated patients
RELATIVE EXCLUSION CRITERIA toward less disability.23 Several recent trials suggest these out-
Older than 80 years old comes can be further improved with careful selection of patients
Severe stroke (NIHSS > 25) for endovascular rescue therapy.
Taking an oral anticoagulant regardless of INR
History of both diabetes and prior ischemic stroke
Endovascular Rescue Therapy
INR, International normalized ratio; IV, intravenous; NIHSS, National Institutes of
Health Stroke Scale; rtPA, recombinant tissue plasminogen activator. The results of several concurrent trials investigating prompt endo-
vascular rescue therapy (usually following IV thrombolysis) versus
medical management were published in late 2014 and early
thrombectomy.20 Less favorable outcomes without thrombolysis 2015. These studies have conclusively demonstrated that patients
have been demonstrated in this population of minor stroke with severe strokes and evidence of proximal large vessel occlu-
patients in particular. Because the risk of symptomatic intra- sions have significantly better functional outcomes when treated
cranial hemorrhage (SICH) is proportional to severity, we recom- with the new generation devices.24-26 Each of these trials used dif-
mend carefully discussing the benefits and risks with patients ferent imaging and clinical selection criteria, although the pres-
when considering rtPA in patients with minor or improving (but ence of a proximal occlusion and prompt treatment, preferably
not cleared) deficits. These patients are at lower risk of complica- within 3 hours, were common to all of the protocols. These trials
tions relative to patients with moderate to severe stroke, yet many emphasize the need for better regionalization of stroke care to
who are not treated still have significant long-term disability.21 reduce the time to definitive reperfusion.27 Unlike acute ischemic
IV tPA is not recommended when the time of stroke onset stroke, non-acute cases of intracranial occlusions are best treated
cannot be ascertained reliably, including strokes recognized on with medical therapy and not permanent intracranial stents.28
awakening. Although the aggregate risk of symptomatic ICH is Intra-arterial thrombolysis currently does not have a defined role
about 6% in trials and observational studies, each individual in the treatment of acute ischemic stroke.

Downloaded for Theo Salean (theosalean@gmail.com) at Sriwijaya University from ClinicalKey.com by Elsevier on January 17, 2020.
For personal use only. No other uses without permission. Copyright ©2020. Elsevier Inc. All rights reserved.
CH APTER 91  Stroke 1253

The recent trials demonstrating clear benefit of endovascular time of onset, concomitant medications, and application of a
thrombectomy come after a decade of negative studies. The dif- prehospital stroke scale.
ference in results is due to a combination of improved devices, Supportive care involving attention to airway management and
emphasis on early intervention, and careful patient selection.29-31 perfusion is of the highest priority. Patients with hemorrhagic
Data from two large trials involving almost 40,000 patients indi- stroke are more likely to have an altered level of consciousness that
cate that early use of aspirin in patients with acute ischemic stroke may rapidly progress to unresponsiveness requiring emergent
who were not treated with a fibrinolytic agent was associated with endotracheal intubation. After intubation, a short-acting medica-
a small but significant reduction in rates of stroke recurrence and tion for sedation should be considered so that a neurologic exami-
mortality. These studies in combination suggest a number needed nation can be repeatedly performed and the findings evaluated.
to treat of 77 (ie, 77 stroke patients would need to be treated with Standard care includes establishing IV access and cardiac moni-
daily aspirin therapy to prevent a poor outcome, such as death, toring. Evaluation of blood glucose and appropriate dextrose and
dependency at discharge, or at 6 months after stroke, in one naloxone administration are essential in any patient with altered
patient). The more recent CHANCE trial found that dual- mental status.
antiplatelet therapy with aspirin and clopidogrel for 90 days sig- Patients seen early after symptom onset are at high risk of
nificantly reduced recurrent stroke (from 11.7% to 8.2%) in ongoing bleeding. Approximately 30% of such patients will have
patients initially presenting with high-risk TIA or mild stroke significant hematoma expansion on presentation, leading to neu-
(NIHSS below 4).32 Aspirin should not be given for the first 24 rologic deterioration and worse outcome. Major therapies aimed
hours to patients who have received a fibrinolytic agent and not at reducing this risk include BP reduction, anticoagulation rever-
until a swallowing study has been performed. sal, and hemostatic therapy.
BP control is commonly performed after ICH. This inter-
Anticoagulation vention is controversial, because hypertension may potenti-
ate further bleeding, but lowering BP in a patient with chronic
The use of therapeutic dosing of low–molecular-weight heparin hypertension may decrease CBF, worsening brain injury. One
(LMWH) or unfractionated heparin is now generally abandoned randomized trial found that lowering systolic BP to below 140
in the routine care of stroke patients; prophylactic dosing to reduced ongoing bleeding, but there was no change in neurologic
prevent venous thromboembolism however is a hospital level outcome, suggesting that this therapy may not provide clinical
quality measure and publically reported.33 It may be that any benefit in unselected patients. A larger trial focused on func-
reduction in risk of subsequent ischemic stroke is balanced by an tional outcomes demonstrated that prompt reduction to a systolic
increased risk of hemorrhagic stroke. To date, no studies have blood pressure (SBP) of 140 mm Hg may slightly reduce dis-
definitively established the efficacy of anticoagulants in the man- ability and was generally safe.34 The current consensus regard-
agement of patients with acute ischemic stroke, and current AHA ing management of ICH is to provide antihypertensive treatment
guidelines recommend against the routine use of heparinoids in with parenteral agents for systolic pressures higher than 160 to
this population.4 However, heparin is sometimes considered by 180 mm Hg or MAP higher than 130 mm Hg. Recommended
vascular neurologists in select patients at high risk for stroke pro- agents include labetalol, esmolol, nicardipine, clevidipine, and
gression, including patients with crescendo TIAs or TIA from a hydralazine.3
cardioembolic source (eg, atrial fibrillation, patients with a high- Many patients are coagulopathic at the time of their ICH; and
grade carotid artery stenosis, patients with posterior circulation for patients on anticoagulants, emergency reversal will theoreti-
TIA, and patients with evolving strokes). Heparin or LMWH is cally minimize the risk of further bleeding, although this is not
often instituted to treat carotid and vertebral artery dissection, backed up by clinical trial evidence. For patients on warfarin,
unless a contraindication such as intracranial extension is present. reversal is achieved using IV vitamin K (10 mg IV or subcutane-
If a dissection is diagnosed and the patient has no symptoms of ously), supplemented with either fresh frozen plasma (FFP) (2 to
ischemia, treatment with antiplatelet therapy alone may be an 4 units) or prothrombin complex concentrate (PCC) (Kcentra 25
option. Heparin therapy should not be initiated in patients with to 50 units/kg depending on INR—dosing varies for other PCCs
suspected endocarditis or in any patient until a CT scan has ruled by formulation).3 Of the new generation oral anticoagulants, at
out intracranial bleeding. this time only dabigatran has a specific antidote (idarucizumab)
proven to reverse coagulant effects in human clinical trials.35 Four-
Intracerebral Hemorrhage factor PCC is likely to be the most quickly available reversal agent
for apixaban and rivaroxaban; hemodialysis will rapidly eliminate
No specific therapy has been demonstrated to substantially dabigatran from the circulation but is not generally practical for
improve the outcome of patients with ICH. One of the strongest patients with serious bleeding.36 Hemostatic therapy (treatment
predictors of mortality when considering ICH patients with com- with procoagulant agents; eg, recombinant activated factor VII in
parable severity is the implementation of early care limitations. patients without baseline coagulopathy) was initially promising;
When health care providers initiate early do not resuscitate (DNR) however, a phase III trial showed no clinical benefit.
orders, patients with otherwise equivalent prognoses are more To consider other iatrogenic coagulopathies, post-thrombolytic
likely to die. As a result, we believe that early prognostication in symptomatic intracranial hemorrhage is a serious complica-
the ED should be avoided. Current evidence supports the benefit tion. Although various protocols have been employed for post-
of aggressive medical care. Patients admitted to a specialized unit, thrombolysis intracranial hemorrhage, a comparative observa-
patients who are expedited to the intensive care unit (ICU), those tional study comparing reversal with FFP or cryoprecipitate versus
admitted on a weekday, and those treated more aggressively conservative management (no reversal) demonstrated poor out-
appear to have better neurologic outcomes.61-63 Therefore, even in comes in both groups. Mortality was slightly higher in the reversal
the absence of therapies specifically proven in phase III trials, group, although this study only included 48 patients.37 Based on
multidisciplinary care by specially trained care teams appears to the limited evidence, at this point in time we would recommend
provide benefit. administering cryoprecipitate (6 to 8 units) for thrombolytic asso-
The patient with suspected ICH requires rapid assessment and ciated ICH that occurs shortly (0 to 3 hours) after alteplase admin-
transport to a care center with rapid neuroimaging and intensive istration. Later ICH (3 hours to days) is unlikely to be associated
care management facilities. Out-of-hospital management is with a persistent coagulopathy, because the fibrinogen depletion
similar to that for ischemic stroke, including determination of following alteplase administration is transient; therefore, reversal

Downloaded for Theo Salean (theosalean@gmail.com) at Sriwijaya University from ClinicalKey.com by Elsevier on January 17, 2020.
For personal use only. No other uses without permission. Copyright ©2020. Elsevier Inc. All rights reserved.
1254 PART III  Medicine and Surgery  |  SECTION Seven  Neurology

agents appear to confer risk without a potential biological benefit interventions, and maintain a core infrastructure, such as a stroke
in this time window. unit and stroke registry. The establishment of PSCs and CSCs is
Finally, the data on treatment of patients with spontaneous intended to improve outcomes for stroke patients by ensuring a
ICH who are on preexisting antiplatelet treatments, such as aspirin high level of coordinated care.
or clopidogrel, is conflicting. A small subgroup of an observa- The most recent level of stroke classification for hospitals is the
tional trial (27 patients) suggested patients receiving a platelet acute stroke-ready hospital (ASRH). These hospitals are typically
infusion within 12 hours of onset had an increased odds of a good smaller facilities with lower stroke patient volumes. An ASRH is
functional outcome.38 At this time, the data do not support plate- capable of establishing the initial stroke diagnosis, as well as
let transfusions in patients with spontaneous ICH being treated providing acute stabilization and treatment. The use of tele-
with antiplatelet agents, unless a new thrombocytopenia (platelet technologies between the ASRH and PSC/CSC will likely serve a
count <30,000) has also developed. pivotal role in support of clinical care. After initial stabilization
For patients with clinical or radiographic evidence of elevated and treatment, stroke patients will frequently be transferred to
ICP, therapies aimed at lowering ICP should be considered. First, PSC or CSC institutions.40
neurosurgical consultation is obtained to evaluate the benefits of It is recommended that patients with symptoms consistent
an external ventricular drain (EVD) placement or hematoma with an acute stroke be transported to emergency facilities capable
evacuation. Pending consultation, a number of medical therapies of initiating fibrinolytic therapy within 1 hour of hospital arrival.
aimed at decreasing ICP are available; however, these interven- At a minimum, this requires emergent CT capabilities, an institu-
tions should not be used prophylactically.3 Hyperventilation can tional “acute stroke protocol,” and availability of a physician
be a temporizing measure pending more definitive treatment. knowledgeable in the use of thrombolytic therapy. Intensive care
Mannitol moves fluid from the intracranial compartment, thereby monitoring and neurosurgery capabilities should be available
reducing cerebral edema. Hypertonic saline (3% or 23.4%) can be within 2 hours of drug initiation, either at the treating hospital or
used as an alternative to mannitol or in combination. Other exper- by helicopter or ground transport to an appropriate health care
imental modalities include barbiturate coma and hypothermia. facility.39
Seizure activity can cause neuronal injury, elevations in ICP, In most cases, once the diagnosis of an acute stroke or stroke
and destabilization of an already critically ill patient. However, syndrome is established and the patient is stabilized, hospitaliza-
there is observational evidence that prophylactic administration tion will be necessary for further evaluation and treatment.
of antiepileptic drugs (AEDs) may be harmful; therefore, AEDs Patients may deteriorate over the first 24 hours and require close
should be reserved for patients with known or suspected seizure, in-hospital monitoring. Most patients can be managed on a
including nonconvulsive seizures.3 general medical or telemetry unit, although there is evidence sug-
Surgical evacuation of the hematoma is not beneficial in most gesting a benefit from admission to a stroke-specific unit. Patients
cases of non-cerebellar ICH. Selected patients with a sizable lobar with large acute hemispheric strokes (associated with increased
hemorrhage that is proximal to the cortical surface and associated risk of herniation) or with significant posterior circulation-related
with progressive neurologic deterioration may benefit from surgi- changes and those treated with a fibrinolytic agent should be
cal drainage. Recent studies have also suggested that minimally monitored in a step-down or ICU for at least 24 hours.
invasive surgery (eg, stereotactic insertion of a catheter) may In many centers, patients require admission to receive a prompt
provide some benefit to selected patients. For patients with cere- evaluation for TIA. However, some centers have developed ED
bellar hemorrhage, surgery can be lifesaving, because the infraten- observation unit protocols or rapid outpatient TIA clinics to
torial space is much more limited and high ICP can disrupt vital ensure an expedited evaluation (see Chapter e6). One commonly
brainstem functions. For this reason, many neurosurgeons will used tool available to help select which TIA patients are at highest
consider emergent surgery for patients with cerebellar hemor- risk (and thus likely should be admitted) versus those at lowest
rhage within 48 hours of onset.
In cases of severe intraventricular hemorrhage or hemorrhages
in the posterior fossae, the normal circulation of cerebrospinal
fluid (CSF) can become interrupted, leading to the development TABLE 91.6 
of hydrocephalus. This condition is characterized by an abnormal
rise in CSF volume. In such cases, neurosurgeons will often place ABCD2 Score for Assessing Stroke Risk in Patients
a ventricular catheter. Some groups infuse thrombolytic agents With a Transient Ischemic Attack
through the catheter to help break up the clot and minimize the
risk of hydrocephalus. RISK FACTOR POINTS
Finally, as with ischemic stroke, general supportive care should Age >60 years old 1
be provided to minimize neuronal injury. This includes treating Initial BP >140/90 mm Hg 1
hyperthermia (such as, with acetaminophen) and treating hyper-
glycemia with insulin. Unilateral weakness 2
SPEECH IMPAIRMENT
DISPOSITION Without weakness 1

Ischemic Stroke and Transient Ischemic Attacks Symptoms 10 to 59 minutes 1


Symptoms ≥60 minutes 2
“Stroke center” definitions have been established, and there is a
national certification process for primary stroke centers (PSCs) History of diabetes 1
and comprehensive stroke centers (CSCs) in the United States. In RESULT
broad terms, institutional certification as a PSC requires the estab-
lishment of a stroke infrastructure (ie, a stroke team, stroke unit, 0 to 3 = Low risk (1% risk of stroke in 48 hours)
patient care protocols, and support services, including CT scan- 4 to 5 = Moderate risk (4.1% risk of stroke in 48 hours)
ning and laboratory testing availability), as well as institutional
≥6 = High risk (8% risk of stroke in 48 hours)
administrative support and strong leadership.4,39 CSCs offer
advanced imaging modalities, perform surgical and endovascular BP, Blood pressure.

Downloaded for Theo Salean (theosalean@gmail.com) at Sriwijaya University from ClinicalKey.com by Elsevier on January 17, 2020.
For personal use only. No other uses without permission. Copyright ©2020. Elsevier Inc. All rights reserved.
CH APTER 91  Stroke 1255

risk (who might therefore be safe for expedited outpatient evalu- The majority of patients with an acute hemorrhagic stroke
ation) is the ABCD2 score (Table 91.6). In addition, most patients should be admitted to an ICU in which specialty consultation is
without a contraindication will be started on antithrombotic available. If this is unavailable at the evaluating institution, the
therapy in the ED after consultation with a neurologist. patient should be transported to an appropriate institution.

KEY CONCEPTS
• Anterior circulation strokes result in contralateral hemiparesis of the • In acute ischemic stroke, the patient and or their families should be
face and body. informed of the risk and benefit of treatment with alteplase.
• Vertebrobasilar strokes result in ipsilateral CN deficits and • Patients with a hemorrhagic stroke on coumadin should be promptly
contralateral hemiparesis. reversed using vitamin K and either FFP or PCC.
• Posterior cerebral artery stroke causes ipsilateral CN III palsy and • Prognosis is worse in acute stroke in the setting of fever,
contralateral homonymous hemianopsia. hypotension, hypoxia, and hyperglycemia.
• Wallenberg’s syndrome (lateral medullary syndrome) causes vertigo, • Carotid Doppler, MRA, or CTA studies are recommended before
Horner’s syndrome, ipsilateral facial numbness, loss of corneal reflex, discharge of a patient with TIA from the ED.
and contralateral loss of pain/temperature. • Overly aggressive BP management should be avoided in patients
• Cervical artery dissection is a common cause of stroke in young with acute ischemic stroke.
patients; TIAs preceding stroke in these patients are often missed. • Accurate identification of the last time a patient was known to be at
• The goal for eligible patients is to receive thrombolytics within 90 his or her neurologic baseline should be documented in all patients
minutes of symptoms onset; the dose of alteplase is 0.9 mg per kg with stroke.
with 10% given as a bolus and the remaining 90% given over • IV alteplase followed by endovascular thrombectomy is
1 hour. recommended in patients with large anterior circulation artery
• Acute ischemic stroke patients receiving alteplase are at risk of occlusion.
developing a spontaneous intracranial hemorrhage; the risk is lowest
in patients with a low stroke score, no hypertension, no diabetes,
and age younger than 70.

The references for this chapter can be found online by accessing the accompanying Expert Consult website.

Downloaded for Theo Salean (theosalean@gmail.com) at Sriwijaya University from ClinicalKey.com by Elsevier on January 17, 2020.
For personal use only. No other uses without permission. Copyright ©2020. Elsevier Inc. All rights reserved.
CH APTER 91  Stroke 1255.e1

REFERENCES
1. Kernan WN, Ovbiagele B, Black HR, et al: Guidelines for the prevention of stroke in 19. Brown M, Burton J, Nazarian D, et al: Clinical policy: use of intravenous tissue
patients with stroke and transient ischemic attack: a guideline for healthcare profes- plasminogen activator for the management of acute ischemic stroke in the emergency
sionals from the American Heart Association/American Stroke Association. Stroke department. Ann Emerg Med 66:322–333, 2015.
45:2160–2236, 2014. 20. Coutts SB, Dubuc V, Mandzia J, et al: Tenecteplase–tissue-type plasminogen activator
2. Mozaffarian D, Benjamin EJ, Go AS, et al: Heart disease and stroke statistics—2015 evaluation for minor ischemic stroke with proven occlusion. Stroke 46:769–774, 2015.
update: a report from the American Heart Association. Circulation 131:e29–e322, 21. Khatri P, Conaway M, Jouhnson K: Ninety day outcome rates of a prospective cohort
2015. of consecutive mild ischemic stroke patients. Stroke 43:560–562, 2012.
3. Morgenstern LB, Hemphill JC, Anderson C, et al: Guidelines for the management of 22. Flynn D, Ford GA, Stobbart L, et al: A review of decision support, risk communica-
spontaneous intracerebral hemorrhage: a guideline for healthcare professionals from tion and patient information tools for thrombolytic treatment in acute stroke: lessons
the American Heart Association/American Stroke Association. Stroke 41:2108–2129, for tool developers. BMC Health Serv Res 13:225, 2013.
2010. 23. Mishra NK, Lyden P, Grotta JC, et al: Thrombolysis is associated with consistent
4. Jauch EC, Saver JL, Adams HP, et al: Guidelines for the early management of patients functional improvement across baseline stroke severity: a comparison of outcomes
with acute ischemic stroke a guideline for healthcare professionals from the American in patients from the Virtual International Stroke Trials Archive (VISTA). Stroke
Heart Association/American Stroke Association. Stroke 44:870–947, 2013. 41(11):2612–2617, 2010.
5. Kissela BM, Khoury JC, Alwell K, et al: Age at stroke: temporal trends in stroke 24. Berkhemer OA, Fransen PSS, Beumer D, et al: A randomized trial of intraarterial
incidence in a large, biracial population. Neurology 79:1781–1787, 2012. treatment for acute ischemic stroke. N Engl J Med 372(1):11–20, 2015.
6. Sarikaya H, da Costa BR, Baumgartner RW, et al: Antiplatelets versus anticoagulants 25. Campbell BCV, Mitchell PJ, Kleinig TJ, et al: Endovascular therapy for ischemic
for the treatment of cervical artery dissection: Bayesian meta-analysis. PLoS ONE stroke with perfusion-imaging selection. N Engl J Med 372:1009–1018, 2015.
8:e72697, 2013. 26. Goyal M, Demchuk AM, Menon BK, et al: Randomized assessment of rapid endo-
7. Zinkstok SM, Vergouwen MDI, Engelter ST, et al: Safety and functional outcome of vascular treatment of ischemic stroke. N Engl J Med 372:1019–1030, 2015.
thrombolysis in dissection-related ischemic stroke: a meta-analysis of individual 27. Fargen KM, Fiorella D, Albuquerque F, et al: Systematic regionalization of stroke care.
patient data. Stroke 42:2515–2520, 2011. J Neurointerv Surg 7(4):229–230, 2015.
8. Saposnik G, Barinagarrementeria F, Brown RD, Jr, et al: Diagnosis and management 28. Derdeyn CP, Chimowitz MI, Lynn MJ, et al: Aggressive medical treatment with or
of cerebral venous thrombosis: a statement for healthcare professionals from the without stenting in high-risk patients with intracranial artery stenosis (SAMMPRIS):
American Heart Association/American Stroke Association. Stroke 42(4):1158–1192, the final results of a randomised trial. Lancet 383:333–341, 2014.
2011. 29. Ciccone A, Valvassori L, Nichelatti M, et al: Endovascular treatment for acute isch-
9. Gee C, Dawson M, Bledsoe J, et al: Sensitivity of newer-generation computed tomog- emic stroke. N Engl J Med 368:904–913, 2013.
raphy scanners for subarachnoid hemorrhage: a Bayesian analysis. J Emerg Med 30. Broderick JP, Palesch YY, Demchuk AM, et al: Endovascular therapy after intravenous
43(1):13–18, 2012. t-PA versus t-PA alone for stroke. N Engl J Med 368:893–903, 2013.
10. Wintermark M, Sanelli PC, Albers GW, et al: Imaging recommendations for acute 31. Saver JL, Jahan R, Levy EI, et al: Solitaire flow restoration device versus the Merci
stroke and transient ischemic attack patients: a joint statement by the American Retriever in patients with acute ischaemic stroke (SWIFT): a randomised, parallel-
Society of Neuroradiology, the American College of Radiology, and the Society of group, non-inferiority trial. Lancet 380:1241–1249, 2012.
NeuroInterventional Surgery. AJNR Am J Neuroradiol 34(11):E117–E127, 2013. 32. Wang Y, Wang Y, Zhao X, et al: Clopidogrel with aspirin in acute minor stroke or
11. Holmes M, Rathbone J, Littlewood C, et al: Routine echocardiography in the manage- transient ischemic attack. N Engl J Med 369:11–19, 2013.
ment of stroke and transient ischaemic attack: a systematic review and economic 33. Fonarow GC, Alberts MJ, Broderick JP, et al: Stroke outcomes measures must be
evaluation. Health Technol Assess 18(16):1–176, 2014. appropriately risk adjusted to ensure quality care of patients: a presidential advisory
12. Poisson S, Johnston SC: Prevention of stroke following transient ischemic attack. from the American Heart Association/American Stroke Association. Stroke 45:1589–
Curr Atheroscler Rep 13:330–337, 2011. 1601, 2014.
13. Jauch EC, Cucchiara B, Adeoye O, et al: Part 11: Adult Stroke: 2010 American Heart 34. Anderson CS, Heeley E, Huang Y, et al: Rapid blood-pressure lowering in patients
Association Guidelines for Cardiopulmonary Resuscitation and Emergency Cardio- with acute intracerebral hemorrhage. N Engl J Med 368:2355–2365, 2013.
vascular Care. Circulation 122(18 Suppl 3):S818–S828, 2010. 35. Pollack CV, Jr, Reilly PA, Eikelboom J, et al: Idarucizumab for dabigatran reversal.
14. Lin CB, Peterson ED, Smith EE, et al: Emergency medical service hospital prenotifica- N Engl J Med 373(6):511–520, 2015.
tion is associated with improved evaluation and treatment of acute ischemic stroke. 36. Kaatz S, Kouides PA, Garcia DA, et al: Guidance on the emergent reversal of oral
Circ Cardiovasc Qual Outcomes 5(4):514–522, 2012. thrombin and factor Xa inhibitors. Am J Hematol 87(Suppl 1):S141–S145, 2012.
15. Wardlaw JM, Murray V, Berge E, et al: Thrombolysis for acute ischaemic stroke. 37. Alderazi YJ, Barot NV, Peng H, et al: Clotting factors to treat thrombolysis-related
Cochrane Database Syst Rev (7):CD000213, 2014. symptomatic intracranial hemorrhage in acute ischemic stroke. J Stroke Cerebrovasc
16. Lees KR, Bluhmki E, von Kummer R, et al: Time to treatment with intravenous Dis 23:e207–e214, 2014.
alteplase and outcome in stroke: an updated pooled analysis of ECASS, ATLANTIS, 38. Naidech AM, Liebling SM, Rosenberg NF, et al: Early platelet transfusion improves
NINDS, and EPITHET trials. Lancet 375(9727):1695–1703, 2010. platelet activity and may improve outcomes after intracerebral hemorrhage. Neuro-
17. Scott PA, Meurer WJ, Frederiksen SM, et al: A multilevel intervention to increase crit Care 16(1):82–87, 2012.
community hospital use of alteplase for acute stroke (INSTINCT): a cluster- 39. Alberts MJ, Latchaw RE, Jagoda A, et al: Revised and updated recommendations for
randomised controlled trial. Lancet Neurol 12:139–148, 2013. the establishment of primary stroke centers: a summary statement from the brain
18. Sandercock P, Wardlaw JM, Lindley RI, et al: The benefits and harms of intravenous attack coalition. Stroke 42(9):2651–2665, 2011.
thrombolysis with recombinant tissue plasminogen activator within 6 h of acute 40. Alberts MJ, Wechsler LR, Jensen ME, et al: Formation and function of acute stroke-
ischaemic stroke (the third international stroke trial [IST-3]): a randomised con- ready hospitals within a stroke system of care recommendations from the brain attack
trolled trial. Lancet 379:2352–2363, 2012. coalition. Stroke 44(12):3382–3393, 2013.

CHAPTER 91: QUESTIONS & ANSWERS


91.1. Which of the following statements concerning ischemic has been a previous contralateral stroke. ACA strokes primarily
stroke is true? affect frontal lobe functions and may also present with primitive
A. Anterior circulation strokes are more likely than grasp and such reflexes. In addition to contralateral motor and
posterior strokes to show evidence of progression at sensory defects, MCA strokes may present with expressive aphasia,
the time of presentation. agnosia, and ipsilateral hemianopsia.
B. Anterior circulation strokes rarely present with
complete loss of consciousness. 91.2. Which of the following is not associated with posterior
C. Posterior cerebral artery strokes are associated with circulation strokes?
incontinence, leg weakness greater than arm A. Diplopia
weakness, and gait clumsiness. B. Homonymous cranial nerve (CN) and extremity
D. The presence of aphasia suggests an anterior cerebral motor deficits
artery (ACA) distribution stroke, typically left sided. C. Loss of consciousness
E. The presence of diplopia suggests an anterior D. Loss of visual object recognition
circulation stroke. E. Nausea, vomiting, and ataxia
Answer: B. Forty percent of posterior and 20% of anterior circula- Answer: B. Posterior circulation (vertebrobasilar) strokes involve
tion strokes present with progressive symptoms. It is rare for the vertebral, basilar, and posterior cerebral arteries. Because
anterior circulation (carotid, ACA, and middle cerebral artery this system supplies the reticular activating system, cerebellum,
[MCA]) strokes to significantly alter consciousness, unless there brainstem, occipital lobe, and brainstem vomiting centers, loss of

Downloaded for Theo Salean (theosalean@gmail.com) at Sriwijaya University from ClinicalKey.com by Elsevier on January 17, 2020.
For personal use only. No other uses without permission. Copyright ©2020. Elsevier Inc. All rights reserved.
1255.e2 PART III  Medicine and Surgery  |  SECTION Seven  Neurology

consciousness with vomiting, visual changes, and cerebral ataxia D. 5


may be seen. Ipsilateral CN deficits (because these nuclei largely E. 6
reside in the brainstem) occur with contralateral “body” deficits
Answer: E. Thus ischemic stroke trials, using fibrinolytic or anti-
resulting from motor/sensory fiber decussation.
platelet agents, have attempted to recanalize occluded arteries and
reperfuse ischemic areas of the brain within a 2- to 6-hour thera-
91.3. Headache, vomiting, and a decreased level of
peutic window.
consciousness are most commonly seen with which of
the following disorders?
91.7. Which of the following areas of the brain is perfused by
A. Ischemic stroke
the posterior circulation?
B. Intracranial hemorrhage
A. Internal capsule
C. Migraine headache
B. Posterior aspect of the temporal lobe
D. Subarachnoid hemorrhage (SAH)
C. Putamen
E. Tic douloureux
D. Speech areas of the temporal lobe
Answer: D. The incidence of headache is highest by far in patients E. Thalamus
with SAH. Vomiting and depressed loss of consciousness are also
Answer: E. The thalamus is perfused by the posterior circulation.
generally more common in this group. Pure migraine headache
The other areas are perfused by the anterior circulation.
rarely, if ever, causes a depressed loss of consciousness. Tic head-
aches do not cause loss of consciousness.
91.8. Which of the following statements regarding stroke
etiology is true?
91.4. What percentage of patients with hemorrhagic stroke
A. Lacunar strokes reliably cause a pure motor deficit.
experience clinical deterioration because of growth in
B. Less than 1% of strokes occur in the 15- to 45-year-
hemorrhage volume within the first hours?
old age group.
A. 10%
C. One-third of ischemic strokes are thrombotic.
B. 20%
D. Strokes resulting from atrial fibrillation likely involve
C. 30%
small vessels.
D. 50%
E. Two-thirds of ischemic strokes are cardioembolic.
E. 75%
Answer: C. One-third of ischemic strokes are thrombotic. Lacunar
Answer: C. Thirty percent of patients with intracerebral hemor-
strokes may cause a pure motor, pure sensory, or ataxic/hemiparesis
rhage (ICH) experience early hemorrhage expansion. Progression
stroke. Vessel occlusion resulting from atrial fibrillation–induced
of neurologic deficits and decreasing mental status suggest the
emboli more likely involves the large vessels. Three percent to 4%
diagnosis.
of ischemic strokes occur in the 15- to 45-year-old age group.
91.5. A 69-year-old male presents with headache, vomiting,
91.9. A 29-year-old female presents with a left-sided headache
aphasia, a right lower facial palsy, and right upper greater
after a moderate-speed motor vehicle collision (MVC).
than right lower extremity weakness. The symptoms
She suffered no loss of consciousness and has no other
began approximately 4 hours before arrival. Vital signs
complaints or obvious injuries. Physical examination is
are temperature 99° C, blood pressure (BP)
remarkable only for drooping of the left eyelid and slight
180/90 mm Hg, respiratory rate 18 breaths per minute,
miosis of the left pupil compared with the right. Which
heart rate 92 beats per minute, and oxygen saturation
of the following would be the diagnostic test of choice?
96% on room air. Emergent computed tomography (CT)
A. Brain magnetic resonance imaging (MRI) with
scan shows a left temporal intracerebral hemorrhage
gadolinium
(ICH). Soon after presentation, the patient experiences
B. Contrasted computed tomography (CT) scan of the
increased vomiting and a diminishing level of
brain
consciousness. What is the most likely explanation for
C. CT angiogram of the carotid arteries
this deterioration?
D. Uncontrasted CT scan of the brain
A. Accompanying subarachnoid hemorrhage (SAH)
E. Urine drug screen
B. Acute brainstem herniation
C. Hypoxia from neurogenic pulmonary edema Answer: C. Carotid or vertebral artery dissection can occur after
D. Increase in volume of the ICH trauma or mild events, such as yoga, twisting, or prolonged static
E. Myocardial infarction with cardiogenic shock positions looking upward. The hallmark is unilateral neck pain,
face pain, or headache, often with accompanying Horner’s syn-
Answer: D. Approximately one-third of patients with ICH experi-
drome. Acutely, cerebral ischemic changes would not be seen on
ence early hemorrhage volume expansion. Although brainstem
brain imaging. Carotid and vertebral dissection is not a contrain-
herniation is a possibility, this is typically a later sequelae with a
dication for thrombolytic therapy in the eligible patient.
more gradual presentation. Acute myocardial infarction may be
associated with intracranial emergencies but would not likely
91.10. What percentage of patients who experience a transient
cause an abrupt mental status change. Neurogenic pulmonary
ischemic attack (TIA) will develop a stroke within 3
edema may accompany any condition with elevated intracranial
months?
pressure (ICP) but, again, would not likely cause an abrupt mental
A. 5%
status change.
B. 10%
C. 15%
91.6. After complete occlusion of cerebral vessels, irreversible
D. 20%
neurologic deficits are expected to reliably occur within
E. 25%
how many hours?
A. 2 Answer: B. One-tenth of them will occur within 2 days of the
B. 3 sentinel event.
C. 4

Downloaded for Theo Salean (theosalean@gmail.com) at Sriwijaya University from ClinicalKey.com by Elsevier on January 17, 2020.
For personal use only. No other uses without permission. Copyright ©2020. Elsevier Inc. All rights reserved.
CH APTER 91  Stroke 1255.e3

91.11. A 28-year-old G3P3 woman who is 2-weeks postpartum D. Seizure at the onset of the stroke
after an uncomplicated vaginal delivery presents with E. Symptoms rapidly improving
acute onset of mild headache, lethargy, and double
Answer: B. In the 3- to 4.5-hour window, patients cannot exceed
vision. Physical examination is remarkable for normal
80 years of age. Heparin administration within the 48 hours pre-
vital signs and a left eye lateral gaze palsy. The most
ceding stroke onset is a contraindication in both the 0- to 3-hour
appropriate intervention is likely to be which of the
window as well as the 3- to 4.5-hour window. Similarly, high clini-
following?
cal suspicion for SAH, seizure at the onset of the stroke symptoms,
A. Computed tomography (CT) scan of the brain with
and rapidly improving symptoms are contraindications to fibri-
possible lumbar puncture
nolysis in both time windows.
B. CT scan of the brain and intravenous (IV) heparin
C. Erythrocyte sedimentation rate (ESR) and IV
91.14. A 66-year-old female presents with a possible transient
corticosteroids
ischemic attack (TIA). Approximately 1 hour before her
D. IV magnesium
arrival, she had a 15-minute episode of strictly right arm
E. Lumbar puncture and IV antibiotics
and right leg weakness, and her symptoms have now
Answer: B. Cerebral venous thrombosis may present with head- resolved. Her blood pressure (BP) is 165/92 mm Hg. Her
ache, lethargy, cranial nerve (CN) deficits, seizures, or even psy- prior medical history is significant for hypertension, high
chiatric complaints. CT scan and/or magnetic resonance imaging cholesterol, and diabetes mellitus. What is her ABCD2
(MRI)/magnetic resonance angiography (MRA) are likely to score?
reveal the diagnosis. Treatment includes heparin. Neurosurgical A. 4
consultation is not useful. Subarachnoid hemorrhage (SAH) B. 5
would not be expected to cause a focal neurologic deficit. Eclamp- C. 6
sia and meningitis would be expected to give characteristic find- D. 7
ings on history and examination. E. 8
Answer: C. The patient’s ABCD2 score is 6 (age >60 years, BP
91.12. Which of the following statements is true regarding
>140/90 mm Hg, unilateral weakness, symptoms lasting 10 to 59
management of acute ischemic stroke?
minutes, and history of diabetes). No speech impairment is
A. Heparin is indicated for patients in whom
reported.
thrombolysis is not an option.
B. If the initial computed tomography (CT) scan shows
91.15. A 72-year-old male presents with an apparent stroke.
a large left middle cerebral artery (MCA) distribution
Computed tomography (CT) imaging of the brain
stroke but no hemorrhage, thrombolysis would be
demonstrates only a hyperdense middle cerebral artery
indicated.
(MCA) sign and no other ischemic changes. He was last
C. Initial blood pressure (BP) greater than
seen neurologically normal 4 hours earlier. Which of the
185/110 mm Hg would contraindicate thrombolytic
following is a contraindication to fibrinolytic therapy in
treatment.
this patient?
D. Mechanical thrombectomy may be indicated up to 6
A. His blood glucose is 372 mg/dL.
hours after stroke onset.
B. His National Institutes of Health (NIH) Stroke Scale
E. No clot retrieval devices have been U.S. Food and
score is 24.
Drug Administration (FDA) approved for acute
C. His platelet count is 110,000/mm3.
ischemic stroke management.
D. His systolic blood pressure (BP) is 180 mm Hg.
Answer: D. Intra-arterial thrombolysis may offer benefit up to 6 E. He takes warfarin daily.
hours past stroke onset. BPs higher than 185/110 mm Hg are not
Answer: E. Any oral anticoagulant treatment (regardless of the
an absolute contraindication to thrombolysis if they can be
patient’s international normalized ratio [INR]) is a contraindica-
lowered to this level with one or two doses of a parenteral agent,
tion to fibrinolysis in the 3- to 4.5-hour treatment window. Other
such as labetalol or enalapril. The Mechanical Embolus Removal
contraindications include an NIH stroke scale score greater than
in Cerebral Ischemia (MERCI) retrieval device was FDA approved
25, platelet count less than 100,000/mm3, blood glucose greater
in 2004, and several newer stent-retriever devices have been
than 400 mg/dL, and systolic BP greater than 185 mm Hg.
approved since then. In recent trials of thrombectomy, improved
outcomes were demonstrated at least to 6 hours; and outcomes
91.16. A 75-year-old man is brought to the emergency
were even better when patients received reperfusion within 4.5
department (ED) for altered mental status. After
hours of onset. Aspirin has a proven benefit in patients who do
computed tomography (CT) imaging of the brain is
not receive tissue plasminogen activator (tPA). There is no proven
performed, he is found to have a large intracerebral
benefit to heparin, although some practitioners use it in cases at
hemorrhage (ICH). Which of the following options is
high risk of stroke progression.
not an appropriate strategy for lowering intracranial
pressure (ICP)?
91.13. An 82-year-old male presents with an apparent stroke.
A. Barbiturate-induced coma
He is rapidly evaluated and determined to be within
B. Hyperthermia induction
the 3- to 4.5-hour window for fibrinolytic therapy.
C. Hypertonic saline administration
Which of the following exclusion criteria applies only
D. Hyperventilation
to the 3- to 4.5-hour criteria and not the 0- to 3-hour
E. Mannitol administration
criteria?
A. Administration of heparin within the 48 hours Answer: B. Hypothermia is an experimental modality for lower
preceding the stroke onset ICP. Hyperventilation can serve as a temporizing measure for
B. Age older than 80 years reducing ICP. Mannitol and/or hypertonic saline can also be
C. High clinical suspicion for subarachnoid hemorrhage administered. Inducing a barbiturate coma is also an experimental
(SAH) modality.

Downloaded for Theo Salean (theosalean@gmail.com) at Sriwijaya University from ClinicalKey.com by Elsevier on January 17, 2020.
For personal use only. No other uses without permission. Copyright ©2020. Elsevier Inc. All rights reserved.

Das könnte Ihnen auch gefallen